Σελίδα 1 από 1

Δύσκολο Όριο

Δημοσιεύτηκε: Παρ Μαρ 13, 2020 9:38 pm
από TrItOs
Να υπολογιστεί το παρακάτω όριο:
 \begin{large} \lim\limits_{n \to + \infty} \bigg( \frac{1^{n} + 2^{n} + \cdots + (n-1)^n + n^{n}}{n^n} \bigg) \end{large}
Μπορεί να γίνει εφαρμογή της συνάρτησης f(x)=x^n , x > 0 , με κάποιο τρόπο , επειδή αυτή η συνάρτηση είναι κυρτή ;
και επιπλέον νομίζω ότι μπορεί να εφαρμοσθεί και το θεώρημα Stolz - Cesaro

Re: Δύσκολο Όριο

Δημοσιεύτηκε: Παρ Μαρ 13, 2020 10:43 pm
από Λάμπρος Κατσάπας
TrItOs έγραψε:
Παρ Μαρ 13, 2020 9:38 pm
Να υπολογιστεί το παρακάτω όριο:
 \begin{large} \lim\limits_{n \to + \infty} \bigg( \frac{1^{n} + 2^{n} + \cdots + (n-1)^n + n^{n}}{n^n} \bigg) \end{large}
Το όριο είναι \dfrac{e}{e-1}.

Μπορούμε να εφαρμόσουμε το παρακάτω γενικό αποτελέσμα (Tannery's theorem):

Έστω ότι η σειρά \displaystyle \sum_{k=1}^{\infty }a_k(n) συγκλίνει για κάθε n.

Επίσης, έστω ότι για κάθε k ισχύει

\triangleright \lim_{n\rightarrow \infty }a_k(n)=a_k,

\triangleright \left |a_k(n) \right |\leq M_k για κάθε n

και η σειρά \displaystyle\sum_{k=1}^{\infty }M_k συγκλίνει.

Τότε \displaystyle\lim_{n\rightarrow \infty }\sum_{k=1}^{\infty }a_k(n)=\sum_{k=1}^{\infty }a_k.

Εδώ θα θεωρήσουμε a_k(n)=0,k\geq n και a_k(n)=\left ( \dfrac{n-k}{n} \right )^n=\left ( 1-\dfrac{k}{n} \right )^n,0\leq k\leq n-1.

Είναι \left | a_k(n) \right |\leq e^{-k}:=M_k και \displaystyle\sum_{k=0}^{\infty }M_k=\dfrac{e}{e-1}.

Υπάρχουν και άλλες λύσεις.

Edit: Διόρθωση τυπογραφικού. Ευχαριστώ τον Σταύρο Παπαδόπουλο για την επισήμανση.

Re: Δύσκολο Όριο

Δημοσιεύτηκε: Παρ Μαρ 13, 2020 10:45 pm
από Tolaso J Kos
TrItOs έγραψε:
Παρ Μαρ 13, 2020 9:38 pm
Να υπολογιστεί το παρακάτω όριο:
 \begin{large} \lim\limits_{n \to + \infty} \bigg( \frac{1^{n} + 2^{n} + \cdots + (n-1)^n + n^{n}}{n^n} \bigg) \end{large}

Βγάζω \frac{e}{e-1}. Απλά σπάστε το σε κλάσματα .. Μετά είναι άπειρο άθροισμα γεωμετρικής προόδου.

Re: Δύσκολο Όριο

Δημοσιεύτηκε: Παρ Μαρ 13, 2020 10:59 pm
από ΠΑΠΑΔΟΠΟΥΛΟΣ ΣΤΑΥΡΟΣ
Tolaso J Kos έγραψε:
Παρ Μαρ 13, 2020 10:45 pm
TrItOs έγραψε:
Παρ Μαρ 13, 2020 9:38 pm
Να υπολογιστεί το παρακάτω όριο:
 \begin{large} \lim\limits_{n \to + \infty} \bigg( \frac{1^{n} + 2^{n} + \cdots + (n-1)^n + n^{n}}{n^n} \bigg) \end{large}

Βγάζω \frac{e}{e-1}. Απλά σπάστε το σε κλάσματα .. Μετά είναι άπειρο άθροισμα γεωμετρικής προόδου.
Τόλη δεν πάει έτσι.
Αν ήταν ετσι τότε
\lim_{n\rightarrow \infty }\frac{1}{n+1}+\frac{1}{n+2}+....+\frac{1}{n+n}=0+0+0....+0=0

Re: Δύσκολο Όριο

Δημοσιεύτηκε: Παρ Μαρ 13, 2020 11:13 pm
από Mihalis_Lambrou
TrItOs έγραψε:
Παρ Μαρ 13, 2020 9:38 pm
Να υπολογιστεί το παρακάτω όριο:
 \begin{large} \lim\limits_{n \to + \infty} \bigg( \frac{1^{n} + 2^{n} + \cdots + (n-1)^n + n^{n}}{n^n} \bigg) \end{large}
Επειδή η \displaystyle{ \left ( 1-\dfrac {a}{n}\right) ^n} αυξάνει προς την e^{-a} έχουμε

\displaystyle{ \frac{1^{n} + 2^{n} + \cdots + (n-1)^n + n^{n}}{n^n}=   \left ( 1-\dfrac {n-1}{n}\right) ^n+\left ( 1-\dfrac {n-2}{n}\right) ^n + ... + \left ( 1-\dfrac {2}{n}\right) ^n+ \left ( 1-\dfrac {1}{n}\right) ^n + 1 }

\displaystyle{ \le e^{n-1}+e^{n-2}+...+e^{-2}+e^{-1}+1\le \sum _{k=0}^{\infty}e^{-k} = \dfrac {e}{e-1} \,(*)}.

Επίσης για κάθε σταθερό k και για n\ge k έχουμε ότι το παραπάνω άθροισμα είναι

\displaystyle{\ge  \frac{(n-k)^n+ \cdots + (n-1)^n + n^{n}}{n^n}= \left ( 1-\dfrac {k}{n}\right) ^n + ... + \left ( 1-\dfrac {2}{n}\right) ^n+ \left ( 1-\dfrac {1}{n}\right) ^n + 1}

Παίρνοντας όριο στο δεξί μέλος παρατηρούμε ότι συγκλίνει στο \displaystyle{e^{-k}+...+e^{-1}+1}. Επειδή το το τελευταίο αυξάνει προς \displaystyle{\dfrac {e}{e-1}} έπεται από ισοσυγκλίνουσες και την (*) ότι το ζητούμενο όριο υπάρχει και είναι ίσο με \displaystyle{  \dfrac {e}{e-1}}.

Re: Δύσκολο Όριο

Δημοσιεύτηκε: Παρ Μαρ 13, 2020 11:24 pm
από ΠΑΠΑΔΟΠΟΥΛΟΣ ΣΤΑΥΡΟΣ
TrItOs έγραψε:
Παρ Μαρ 13, 2020 9:38 pm
Να υπολογιστεί το παρακάτω όριο:
 \begin{large} \lim\limits_{n \to + \infty} \bigg( \frac{1^{n} + 2^{n} + \cdots + (n-1)^n + n^{n}}{n^n} \bigg) \end{large}

Είναι
\displaystyle \left ( \dfrac{n-k}{n} \right )^n=\left ( 1-\dfrac{k}{n} \right )^n \leq e^{-k}

για 0 \leq k\leq n.
και
\displaystyle \left ( 1-\dfrac{k}{n} \right )^n \rightarrow e^{-k}
(είναι αύξουσα η ακολουθία)
Ετσι
\displaystyle a_{n}=\frac{1^{n} + 2^{n} + \cdots + (n-1)^n + n^{n}}{n^n}\leq \sum_{k=0}^{n-1}e^{-k}\leq \sum_{k=0}^{\infty }e^{-k}=\frac{e}{e-1}

Από την τελευταια παίρνουμε
\displaystyle limsup a_{n}\leq \frac{e}{e-1}(1)

Εστω k σταθερό.
Για n>k
είναι
\displaystyle a_{n}=\frac{1^{n} + 2^{n} + \cdots + (n-1)^n + n^{n}}{n^n}\geq 1+(1-\frac{1}{n})^{n}+(1-\frac{2}{n})^{n}+......+(1-\frac{k}{n})^{n}

Από την τελευταία παίρνουμε ότι
\displaystyle liminf a_{n}\geq \sum_{m=0}^{k}e^{-m}

Αυτή ισχύει όμως για κάθε k .
Αρα θα είναι
\displaystyle liminf a_{n}\geq \sum_{m=0}^{\infty }e^{-m}=\frac{e}{e-1}(2)
Από (1),(2) έχουμε το όριο.